PTB 1-15

Ace your homework & exams now with Quizwiz!

During the change-of-shift report, the assigned nurse notes a client of the Catholic religion is scheduled to be admitted for the delivery of a ninth child. Which comment made by a nurse indicates an attitude of prejudice? "I think she needs to go to the city hospital." "I guess she doesn't understand how to use birth control." "All those people indulge in large families!" "I wonder who is paying for this trip to the hospital?"

"All those people indulge in large families!" Prejudice is a hostile attitude toward individuals simply because they belong to a particular group presumed to have objectionable qualities. Prejudice refers to preconceived ideas, beliefs, or opinions about an individual, group or culture that limit a full and accurate understanding of the individual, culture, gender, race, event or situation.

A 35 year-old female client talks to the nurse in her health care provider's office about her new diagnosis of uterine fibroids. What statement by the woman is incorrect and indicates that more teaching is needed? "Even if the fibroids cause no problems, they will still need to be taken out." "I sometimes experience pelvic pressure and pain, heavy menstrual bleeding, and I feel the need to urinate more often." "Fibroids occur more frequently in women my age but no one knows what causes them." "Uterine fibroids are noncancerous tumors that grow slowly."

"Even if the fibroids cause no problems, they will still need to be taken out." Fibroids that cause no findings may require only "watchful waiting." The client may just need pelvic exams or ultrasounds every once in a while to monitor the fibroid's growth. Treatment for the symptoms of fibroids (such as painful menses and heavy periods) may include oral contraceptives, IUDs, iron supplements to prevent or treat anemia (due to heavy periods), NSAIDs for cramps or pain or even short-term hormonal therapy to help shrink the fibroids. Surgical removal using myomectomy or hysterectomy is usually reserved as a final alternative after other treatment options have failed to provide adequate relief. In addition, concerns about loss of fertility with this diagnosis and its treatment may be important to this client who is still in her childbearing years.

A nurse who cares for clients undergoing treatment for cancer might expect clients diagnosed with cancer to make the following statements. Based on an understanding of the stages of the grieving process, place the statements in the correct order. "I don't know where to go or what to do." "I will just go on with my life." "If I eat a more balanced diet, I can live longer." "I think the tests got mixed up." "I am so mad at everyone for always reminding me that I have it."

"I think the tests got mixed up." "I am so mad at everyone for always reminding me that I have it." "If I eat a more balanced diet, I can live longer." "I don't know where to go or what to do." "I will just go on with my life." The phases of loss or the grief process according to Dr. Kubler-Ross are: denial, anger, negotiation, depression and acceptance.

Two hours after receiving the first does of lithium (Eskalith, Lithobid), the client reports fine hand tremors. What is the nurse's best explanation for these findings? "You are probably having an allergic reaction. The medication should be discontinued." "Taking lithium on an empty stomach should help minimize these symptoms." "These are common and expected side effects and should subside in a few days." "Reducing dietary intake of sodium and fluids should minimize any side effects."

"These are common and expected side effects and should subside in a few days." Tremors are common side effects that usually subside quickly. Informing clients of these possible reactions can help them tolerate these initial difficulties, while continuing to take the drug and obtaining therapeutic effects.

At the client's request, the nurse performs a fingerstick to test the client's blood glucose and the results are 322 mg/dL. Following standing orders, the nurse administers 3 units of insulin lispro (Humalog) at 11 am. When does the nurse anticipate the insulin lispro will begin to act? 12 noon 3:00 pm 11:15 am 1:00 pm

11:15 am The onset of action and peak for insulin lispro, which is a rapid-acting insulin, is 10 to 15 minutes after administration. This type of insulin will peak in about 1.5 to 2.5 hours. It is designed to cover meals and lower high blood sugar readings.

The order is for ibuprofen (Motrin) oral drops 10 mg/kg of body weight. The client weighs 62 lbs. Motrin oral drops are supplied in bottles containing 40 mg/mL. How many milliliters will the nurse administer? (Report to the nearest whole number.) mL.

7 X mL = 1 mL/40 mg X 10 mg/kg X 1 kg/2.2 lbs X 62 lbs = 620/88 = 7.05 or 7 mL Ratio : 62 lbs/x = 1 kg/2.2 lbs = 28.19 kg 10 mg/x = 1 mL/40 mg = 10/40 = 0.25 0.25 X 28.19 = 7.05 or 7 mL

An RN from the women's wellness health clinic is temporarily reassigned to an adult medical-surgical unit. Which of these client assignments would be appropriate for this nurse? A newly diagnosed client with type 2 diabetes mellitus who is learning foot care A newly admitted client with a diagnosis of pancreatic cancer and severe dehydration A client from a motor vehicle accident with an external fixation device on the leg A client admitted for a barium swallow after a transient ischemic attack (TIA)

A client from a motor vehicle accident with an external fixation device on the leg This client is the most stable, requires basic safety measures and has a predictable outcome. The clues in the other options that indicate to not delegate to a reassigned nurse are: "newly diagnosed," "after a TIA," and "newly admitted... severe dehydration." All of these client have an illness as compared to the client who is healthy except for a fracture from an accident.

An internal disaster has occurred at the hospital. Which of these clients would the nurse recommend to stay in the facility? An older adult client admitted two days ago with an acute exacerbation of ulcerative colitis A young adult in the second day of treatment for an overdose of acetaminophen (Tylenol) A middle-aged client known to have had an uncomplicated myocardial infarction four days ago An adolescent diagnosed with sepsis seven days ago and whose vital signs are maintained within low normal limits

A young adult in the second day of treatment for an overdose of acetaminophen (Tylenol) An overdose of acetaminophen (Tylenol) requires close observation for three to four days as well as Mucomyst by mouth during that time. A strong risk for liver failure exists after an overdose of Tylenol.

A nurse is teaching an older adult client to use a metered-dose inhaler (MDI) and is concerned that the client is unable to coordinate the release of the medication with the inhalation phase. What should the nurse recommend to improve the delivery of the medication? Request a visiting nurse to follow the client at home Add a spacer device to the MDI canister Ask a family member to assist the client with the MDI Nebulized treatments for home care

Add a spacer device to the MDI canister Use of a spacer is especially useful with older adults because it allows more time to inhale and requires less eye-hand coordination. If the client is not using the MDI properly, the medication can get trapped in the upper airway with an outcome of a dry mouth and throat irritation. Using a spacer will allow more drug to be deposited in the lungs and less in the mouth.

A nurse admits a client transferred from the emergency department (ED). The client, diagnosed with a myocardial infarction, reports substernal chest pain, diaphoresis and nausea. What should be the first action taken by the nurse? Order the PRN 12-lead ECG Obtain vital signs Give PRN sublingual nitroglycerin Administer intravenous morphine sulfate as ordered

Administer intravenous morphine sulfate as ordered Decreasing the client's pain is the priority at this time. As long as pain is present, a danger exists for the extension of the infarcted area. Morphine administered intravenously or sublingually will quickly decrease the oxygen demands of the heart, dilate the coronary arteries and cause the client to relax, further decreasing myocardial oxygen demand. Because the client is diagnosed with an MI, the narcotic analgesic should be given instead of the nitroglycerin, which is used to dilate the coronary arteries in ischemic episodes such as acute coronary syndrome. The other actions are also appropriate, but are a lower priority than the immediate relief of ischemic pain.

A client is admitted with severe injuries resulting from an auto accident. The client's vital signs are BP 120/50, pulse rate 110, and respiratory rate of 28. What should be the initial nursing intervention? Initiate the ordered intravenous therapy Institute continuous cardiac monitoring Initiate continuous blood pressure monitoring Administer oxygen as ordered

Administer oxygen as ordered Early findings of shock are associated with hypoxia and manifested by a rapid heart rate and rapid respirations. Therefore, oxygen is the most critical initial intervention; the other interventions are secondary to oxygen therapy.

The nurse is assessing a client with a stage II skin ulcer. Which of these approaches should be most effective to promote healing? Apply a hydrocolloid or foam dressing Use hydrogen peroxide soaks Leave the area open to dry Cover the wound with a dry dressing.

Apply a hydrocolloid or foam dressing In prior years an accepted treatment was the use of a transparent cover. However, evidence-based nursing outcomes now recommends the use of the foam (DuoDerm) dressings to keep the wound somewhat moist. One could eliminate the two options that have the word "dry" in them. This is called the elimination of "similar - dissimilar" options. Hydrogen peroxide should not be used full strength on any type of wound because it is destructive to the cells.

An 8 year-old child is brought to the clinic by a parent who states: "This child was so sick last year with cold and the flu. I want her to have the influenza vaccine this year." The nurse assesses the child and reviews the child's history. What information would the nurse recognize as a contraindication for giving the child this vaccine? Persistent, inconsolable crying after receiving other immunizations Family history of convulsions Allergy to eggs Recent exposure to an infectious disease

Allergy to eggs An allergy to egg proteins is listed by the CDC as a contraindication for administering the influenza vaccine. Influenza vaccines are grown on egg embryos and may contain a small amount of egg protein.

Which is the appropriate injection site to give an influenza vaccine to an adult? *picture of arm*

An influenza (or flu) vaccine is less than 1 mL of fluid, so it is acceptable to administer this injection in the deltoid muscle. The deltoid muscle of the upper arm is a site that is easily accessible in public settings where mass vaccinations are administered.

A client is admitted for first- and second-degree burns on the face, neck, anterior chest and hands. What should be the nurse's priority action? Cover the areas with dry sterile dressings Administer pain medication Initiate intravenous therapy Assess for dyspnea or stridor

Assess for dyspnea or stridor Due to the location of the burns, the client is at risk for the development of upper airway edema and subsequent respiratory distress. The other options are correct, but the priority is to assess breathing and manage the airway. The client with any signs of airway injury will be intubated and given 100% oxygen.

While caring for a client, the nurse notes a pulsating mass in the client's periumbilical area. Which of these assessments is appropriate for the nurse to perform on the mass? Measure the length Auscultate Percuss Palpate

Auscultate Auscultation of the abdomen and the finding of a bruit would confirm the presence of an abdominal aneurysm. This would form the basis of information to be given to the health care provider. The mass should not be palpated or percussed because of the risk of rupture.

The nurse is caring for a child who is diagnosed with coarctation of the aorta. Which finding would the nurse expect when assessing the child? Strong pedal pulses Normal femoral pulses Bounding pulses in the arms Diminished carotid pulses

Bounding pulses in the arms Coarctation of the aorta, which is a narrowing or constriction of the descending aorta, causes increased blood flow to the upper extremities, resulting in a bounding pulse in the arms. Cardinal signs include resting systolic hypertension, absent or diminished femoral and pedal pulses, and a widened pulse pressure.

The 72 year-old client has an estimated blood loss of 600 mL during a gastric resection. The surgeon orders two units of packed cells (PC) to be administered in the post anesthesia care unit. During the administration of the second unit of PC, the nurse notes the following findings: hypertension, a bounding pulse, and increasing dyspnea. What is the probable cause of these findings? Circulatory overload Hemolytic transfusion reaction Transfusion-associated graft-versus-host disease Bacterial transfusion reaction

Circulatory overload Older clients are at risk for circulatory overload, especially when solutions are administered rapidly. Hypertension with a bounding pulse and dyspnea are key signs of fluid overload. The nurse should stop the infusion and contact the health care provider. The other options are related to blood transfusion reactions but are not related to findings of circulatory overload.

Following a surgical procedure, a pneumatic compression device is applied to the adult client. The client reports that the device is hot and the client is sweating and itching. Which of the following steps should the nurse take? (Select all that apply.) Collaborate with health care provider for anti-embolism stockings to be worn under the sleeves of the device Confirm pressure setting of 45 mm Hg Check for appropriate fit Explain that the health care provider ordered the device and it cannot be removed Inform the client that removing the device will likely result in the formation of deep vein thrombosis

Collaborate with health care provider for anti-embolism stockings to be worn under the sleeves of the device Confirm pressure setting of 45 mm Hg Check for appropriate fit In any situation in which a client has discomfort associated with a medical device, the nurse should ensure it is applied correctly and functioning safely. The usual safe and effective pressure range is 35 to 55 mm Hg. Explanations to the clients should support their informed decision-making capabilities and should not be phrased to intimidate or remove client autonomy. Applying anti-embolism stockings under the disposable sleeves of the device may help with the sweating and itching.

A nurse is preparing a client for discharge following inpatient treatment for pulmonary tuberculosis (TB). Which of these instructions should be given to the client? Take medication with aluminum hydroxide (Amphojel) if epigastric distress occurs Avoid contact with children, pregnant women or immunosupressed persons Continue taking medications as prescribed Continue taking medications until findings are relieved

Continue taking medications as prescribed Early cessation of treatment may lead to development of drug resistant TB. Active TB is usually treated with a combination of four different antibiotics (isoniazid, rifampin, ethambutol and pyrazinamide) and can take anywhere from 6 to 12 months to completely kill the bacteria. As with any antibiotics, clients should continue to take medications even after they begin to feel better. There is no reason to avoid contact with children, pregnant women or immunosupressed persons once discharged from the hospital and adhering to medication schedule. Isoniazid should be taken on an empty stomach; ethambutol can be taken with food to avoid stomach upset. If taken with TB medications, Amphojel will interfere with absorption of these medications.

A nurse is caring for a postoperative client who develops evisceration of the abdominal incision. Which intervention should the nurse implement first to prevent additional complications? Cover the wound with a sterile saline-soaked dressing Medicate the client for pain with PRN order Call the health care provider within the hour Place the client in dorsal recumbent position

Cover the wound with a sterile saline-soaked dressing When evisceration occurs, the wound should first be quickly covered by sterile saline soaked dressings. This prevents tissue damage and drying of the area until a surgical repair can be done. The other interventions are also appropriate, though the call to the provider should occur immediately, as this is a medical emergency.

The parents of a 6 year-old child who normally enjoys school tells a nurse that the child has not been doing well since a grandmother died two months ago. Which statement most accurately describes thoughts on death and dying at this age? The child feels guilty for the grandmother's death Death is perceived as being irreversible Death is personified as the bogeyman or devil The child is worried that the child, too, might die

Death is personified as the bogeyman or devil Personification of death is typical of this developmental level. Recall that this age is at the end of the preschool period where magical thinking for the animation of inanimate objects is present.

The nurse is teaching effective stress management techniques to a client one hour before surgery. Which of these actions should the nurse recommend? Imagery Distraction Biofeedback Deep breathing

Deep breathing is a reliable and valid method for stress reduction and can be taught and reinforced in a short period of time preoperatively. The other approaches require more time and repetition over time for maximum effectiveness.

A nurse, who is assigned for five days to a client who has exhibited manipulative behaviors, becomes aware of feeling reluctance to interact with the client. The nurse should take what action next? Talk with the client about the negative effects of manipulative behaviors on other clients and staff within the next few days Limit contacts with the client to avoid reinforcement of the manipulative behavior during the work times Develop a behavior modification plan for the client that will promote more functional behavior within the next week Discuss the feelings of reluctance with an objective peer or supervisor within the next 24 hours

Discuss the feelings of reluctance with an objective peer or supervisor within the next 24 hours This is the correct response The nurse who experiences stress in a therapeutic relationship can gain objectivity through discussion with other professionals. The nurse may wish to have a peer observe the nurse-client interactions with this client for a shift and then have a debriefing of positive and negative actions. The nurse must attempt to discover attitudes and feelings in the self that influence the nurse-client relationship in positive and negative ways.

The nurse admitting an 80 year-old client with Alzheimer's disease to a memory care facility is conducting a medication assessment. The nurse would question the use of which of the following alternative and complementary therapies in this client? Coenzyme Q10 Milk thistle Echinacea Ginkgo biloba

Echinacea Echinacea and ginseng are not reported to enhance memory or delay Alzheimer's disease. Echinacea is used to support immune function. Ginseng is used as a stimulant to improve energy. This supplement may contribute to sleep-wake cycle abnormalities common in Alzheimer's, and may worsen blood pressure and heart problems in the older adult. The other substances may have some benefit for cognition and/or may protect against Alzheimer's disease.

A client is about to have an intravenous pyelogram (IVP). After the contrast material is injected, which client reaction should be acted upon immediately by a nurse? Feeling of excessive warmth Hives with severe itching over the body Excessive salty taste in the mouth Face turning a deep ruddy red color

Hives with severe itching over the body Hives over the body with severe itching is a sign of anaphylaxis and should be acted upon with the administration of epinephrine (adrenaline) immediately. The other reactions are considered normal after the dye injection. Prior to any dye injection procedure clients should be informed that these symptoms may occur.

A nurse is assessing a client's home in preparation for discharge. Which of these observations should be given priority consideration? Proximity to emergency services Financial status overall Family's understanding of the client's needs Location of bathrooms

Family's understanding of the client's needs The degree and depth of the family's understanding of the needs for the client is a priority. In addition, functional communication patterns between family members are fundamental to meeting the needs of the client and family.

A hospitalized 8 month-old infant is receiving digoxin (Lanoxin) for the treatment of tetralogy of Fallot. Prior to administering the next dose of medication, the parent reports that the baby vomited one time, just after breakfast. The heart rate is 72 BPM. What should be the initial response of the nurse? Double the next dose Give the dose after lunch Hold the medication Reduce the next dose by half

Hold the medication Toxic side effects of digoxin include bradycardia, dysrhythmia, nausea, vomiting, anorexia, dizziness, headache, weakness and fatigue. It isn't necessary to hold the medication for infants and children if there is only one episode of vomiting. However, it is appropriate to hold the medication and notify the health care provider for bradycardia. Normal resting heart rate for infants 1-11 months-old is 100-160 BPM.

Parents of a 4 year-old boy have just been informed that their son has a congenital neurologic demyelinating disorder that is terminal. The nurse anticipates their reaction to be in which phase of the crisis process? Impact phase Pre-crisis phase Crisis phase Resolution phase

Impact phase There is no data to determine their response phase except the time frame of recent bad news. The impact of crisis is indicative of high levels of stress, sense of helplessness, confusion, disorganization, and the inability to apply problem-solving behavior.

A nurse is providing a parenting class to individuals living in a community of older homes that were built prior to 1978. During a discussion about formula preparation, which statement is the most important by the nurse to tell the parents how to prevent lead poisoning? Use ready-to-feed commercial infant formula Buy bottled water labeled "lead free" to mix with the formula Let tap water run for two minutes before adding to formula concentrate Boil the tap water for 10 minutes prior to preparing the formula

Let tap water run for two minutes before adding to formula concentrate The use of lead-contaminated water to prepare infant formula is a major source of lead poisoning in infants who live in older houses. Drinking water may become contaminated by lead from old lead pipes or the lead solder used in sealing the water pipes in homes prior to 1978. Letting tap water run for several minutes will diminish the risk for lead contamination. These same houses have the risk of lead contamination from paint chips because prior to that time, paint and gasoline contained lead.

A nurse listens to report on an 80 year-old client diagnosed with middle-stage Alzheimer's disease. What information suggests the nurse should do an immediate follow-up, rather than delegate any care to an unlicensed assistive person (UAP)? A 10 mm Hg drop in systolic blood pressure Minimal response to voice and touch of the peripheral areas of the body Increase in heart rate by 10 beats per minute Change in the color of the fingertips from pink to pallor

Minimal response to voice and touch of the peripheral areas of the body A change in level of consciousness (LOC) indicates an acute change in the client's condition and is probably unrelated to the client's diagnosis. A client in the middle stages of Alzheimer's disease is still responsive to environment and to stimuli and the nurse should expect a response that is greater than "minimal to voice and touch." This change would require a detailed assessment by the nurse. Recall that the RN cannot delegate any task to the LPN or UAP unless the client's condition is stable and/or there's a low likelihood of an emergency. A change in the LOC indicates this client is not stable.

The client needs to be moved up in bed. The client is able to partially assist and weighs 135 pounds. Which action by the nursing staff best supports an awareness of ergonomics and safe client handling? (Select all that apply.) Move the bed into the flat position Pull the client up from the head of the bed Use a friction-reducing device Adjust the height of the bed for caregivers Coordinate lifting the client by counting to 3

Move the bed into the flat position Use a friction-reducing device Adjust the height of the bed for caregivers The algorithm for safe client handling and repositioning a client from side-to-side or up in bed states: use 2 to 3 caregivers for a client who can partially assist and who weighs less than 200 pounds, use a friction-reducing device, move the bed so that it's flat and at a comfortable height for the caregivers. The client should not be pulled from the head of the bed. There really is no safe method to manually lift another adult.

A nurse is caring for a client who is diagnosed with Hodgkin's disease and is scheduled for radiation therapy. The nurse recognizes that, as a result of the radiation therapy, the client is most likely to experience which difficulty? Nausea High fever Night sweats Face and neck edema

Nausea As a result of radiation therapy, which is at the lymph nodes throughout the body, nausea often results. Night sweats are a finding in this disease process. These clients are not likely to have a high fever because the lymphatic or immune system is not fully functional.

The nurse observes a client who is exhibiting seizure activity. Which observation is most important in providing information that will enhance further direction of treatment? Note the time from beginning to end Identify the pattern of breathing Determine if loss of bowel or bladder control occurs Observe the sequence and types of movement

Observe the sequence and types of movement It is a priority to note, and then record, what movements are seen during a seizure because the diagnosis and subsequent treatment are often determined by the type of seizure activity. This question is asking for a priority so ALL of the options are correct.

A client asks the nurse to call the police and states: "I need to report that I am being abused by a nurse." The nurse should take which action? Obtain more details of the client's claim of abuse by a nurse Assist with the report of the client's complaint to the police Document the statement on the client's chart with a report to the manager Focus on reality orientation to time, place and person

Obtain more details of the client's claim of abuse by a nurse The advocacy role of the professional nurse, as well as the legal duty of the reasonable prudent nurse, requires the investigation of claims of abuse or violation of rights. The nurse is legally accountable for actions delegated to others. The application of the nursing process requires that the nurse gather more information, assessment before interventions and before documenting or reporting the complaint.

The nurse is assigned to a client newly diagnosed with active tuberculosis (TB) and a productive cough. Which of these interventions would be a priority for the nurse to implement? Place client in a private room and implement droplet precautions Reinforce that everyone should wash their hands before and after entering the room Have the client cough into a tissue and dispose of the tissue in a separate bag Place the client in a negative pressure private room and have disposable particulate respirators available for hospital employees

Place the client in a negative pressure private room and have disposable particulate respirators available for hospital employees A client with active tuberculosis should be hospitalized in a negative pressure room to prevent cross contamination of the disease. The client would be placed on airborne precautions because this bacteria can be suspended in the air for long periods of time and may be carried for long distances on air currents. Any hospital employee entering the room would need to wear a disposable micron mask or disposable particulate respirator (N-95, for example). The Centers for Disease Control and Prevention (CDC) state that visitors can wear surgical masks.

A nurse, during an assessment of a day-old newborn, notices that the breasts are enlarged bilaterally with a white, thin discharge. What action should the nurse perform next? Obtain fluid to check for glucose by Dextrostix Record the findings while thinking that they are "normal" Ask about medications taken during pregnancy Notify the health care provider within that shift

Record the findings while thinking that they are "normal" Newborn infants of both sexes may have engorged breasts and may secrete milk during the first few days to weeks after birth.

A 25 year-old client, who is unresponsive after diagnosis of traumatic brain injury, is to be transferred from the hospital to a long-term care facility (LTC) today. To which staff member should the charge nurse assign this client? Unlicensed assistive personnel (UAP) Nursing student in final semester before graduation Registered nurse (RN) Licensed practical nurse (LPN)

Registered nurse (RN) The RN is responsible for coordination of discharge to home or other agencies. The transfer to a LTC facility often requires referrals.

The nurse is caring for a client in a home setting. Which action is most likely to ensure the safety of the nurse during a home visit? Carry a cell phone, pager and/or hand-held alarm for emergencies Remain alert at all times and leave if cues suggest that the home is not safe Observe no evidence of weapons in the home during the visit Review the client's record for any previous entries about violence prior to the visit

Remain alert at all times and leave if cues suggest that the home is not safe No person or equipment can guarantee nurses' safety, although the risk of violence can be minimized. Before making initial visits, review referral information carefully and have a plan to communicate with agency staff. Schedule appointments with clients. When driving into an area for the first time, note potential hazards and sources of assistance. Become acquainted with neighbors. Be alert and confident while parking the car, walking to the client's door, making the visit, walking back to the car, and driving away. LISTEN to clients. If they tell you to leave, do so.

The nurse observes a client with a diagnosis of obsessive-compulsive disorder on an inpatient unit. Which behavior is consistent with this medical diagnosis? Preference for consistent caregivers Repeatedly checking that a door is locked Repetitive, involuntary movements Verbalized suspicions about thefts on the unit

Repeatedly checking that a door is locked Behaviors that are repeated are consistent with the diagnosis of obsessive-compulsive disorders. These behaviors, performed to reduce feelings of anxiety, often interfere with normal function and attendance at the place of employment. Verbalized suspicions reflect a paranoid thought process. Repetitive, involuntary movements are characteristic of some antipsychotic medication side effects. They are termed extrapyramidal effects such as tardive dyskinesia.

A nurse has been teaching adult clients about cardiac risks when they visit the hypertension clinic. Which evaluation data would be the best measure of learning? Completion of a mailed survey Reported behavioral changes Performance on written tests Responses to verbal questions

Reported behavioral changes If the clients alter behaviors such as smoking, drinking alcohol and stress management, these changed behaviors suggest that learning has occurred. Additionally, physical assessments, observed behaviors and lab data may confirm risk reduction.

As a client is being discharged following resolution of a spontaneous pneumothorax, the client tells a nurse, "I'm going to Hawaii for a vacation next week." The nurse should warn the client to avoid which activity? Swimming Parasailing Scuba diving Surfing

Scuba diving The nurse would strongly emphasize the need for the client with a history of spontaneous pneumothorax problems to avoid high altitudes, flying in unpressurized aircraft and scuba diving. The negative pressure associated with diving could cause the lung to collapse again.

While assessing an Rh positive newborn whose mother is Rh negative, the nurse should recognize the risk for hyperbilirubinemia. Which of these lab results should be reported immediately? Serum bilirubin of 12 mg Positive Coombs test Jaundice evident at 26 hours Hematocrit of 55%

Serum bilirubin of 12 mg The elevated bilirubin is in the range that requires immediate intervention, such as phototherapy. At a serum bilirubin of 12 mg, the neonate is at risk for the development of kernicterus and bilirubin encephalopathy. The health care provider should determine the appropriate therapy after a review of all laboratory findings plus assessment of the newborn

A nurse is speaking at a community meeting about personal responsibility for health when a participant asks about chiropractic treatment for illnesses. What should be the focus of the nurse's response? Mind-body balance Exercise of joints Spinal column manipulation Electrical energy fields

Spinal column manipulation The theory underlying chiropractic treatment is that interference with transmission of mental impulses between the brain and body organs produces diseases. Such interference is caused by misalignment of the vertebrae. Manipulation reduces the misalignment (subluxation).

A client has just returned to the medical-surgical unit postop for a segmental lung resection. After assessing the client, which is the first action the nurse should take? Suction excessive tracheobronchial secretions Administer the prn pain medication Assist the client to turn, deep breathe and cough Monitor oxygen saturation with the application of an oximeter

Suction excessive tracheobronchial secretions Suctioning the excessive tracheobronchial secretions that are present in post-thoracic surgery clients is the priority to maintain an open airway. The application of the pulse oximeter would be next with pain medication given and the mobility last.

The nurse is caring for a client diagnosed with anemia and confusion. Which task could the nurse delegate to the unlicensed assistive personnel (UAP)? Report mental status changes and level of mental clarity Test stool for occult blood and urine for pH with a report of the results Assess and document skin turgor and skin color changes Suggest foods that are high in iron and prepare a list of likes and dislikes

Test stool for occult blood and urine for pH with a report of the results The UAP can perform routine tasks that have known or expected outcomes. These tasks typically do not require nursing judgment or decision-making.

A client diagnosed with testicular cancer has undergone a unilateral orchiectomy. The client expresses fears about his prognosis prior to discharge. What information would the nurse want to include when helping the client better understand this type of cancer? This surgery causes impotence and infertility With early intervention, the cure rate for teticular cancer is about 50% Intensive chemotherapy is the treatment of choice following surgery Testicular cancer has a five-year survival rate of 95% with early diagnosis and treatment

Testicular cancer has a five-year survival rate of 95% with early diagnosis and treatment With aggressive treatment and early detection/diagnosis the cure rate is 90%. The other options are incorrect information. After unilateral orchiectomy, the remaining testicle can produce adequate sperm for fertility and impotence is unlikely. In bilateral orchiectomy, fertility is lost, so sperm banking prior to surgery is recommended. Dissection of lymph nodes for surgical cancer treatment may cause nerve injury, which would increase the risk of impotence.

The nurse receives a report on a client being admitted with the diagnosis of cirrhosis of the liver and ascites. What should the nurse emphasize to the nursing assistant about providing care for this client? The client may ambulate and sit in a chair as tolerated The client is to ambulate as tolerated and be positioned in semi-Fowler's position when in bed The client should remain on bed rest in a semi-Fowler's position The client should ambulate as tolerated, resting in bed with legs elevated between walks

The client should ambulate as tolerated, resting in bed with legs elevated between walks Encourage alternating periods of ambulation and bed rest with legs elevated to mobilize edema and ascites. Encourage and assist the client to gradually increase the duration and frequency of walks.

The parents of a 7 year-old tell the nurse that their child has started to "tattle" on siblings. In interpreting this new behavior, how should the nurse explain the child's actions to the parents? Insecurity and attention getting are common motives The ethical sense and feelings of justice are developing Complex thought processes help to resolve conflicts Attempts to control the family using new coping styles

The ethical sense and feelings of justice are developing The child is developing a sense of justice and a desire to do what is right. At 7, children are increasingly aware of family roles and responsibilities. They also do what is right because of parental direction or to avoid punishment. This age group, 6 to 12 years of age, is called the school-aged group.

The nurse is providing discharge teaching to the parents of a 15 month-old child diagnosed with Kawasaki disease (mucocutaneous lymph node syndrome or infantile polyarteritis). The child has received immunoglobulin therapy. Which instruction point would be appropriate to include during the discussion? Complete recovery is expected within several days The measles, mumps and rubella vaccine should be delayed Active range of motion exercises should be done frequently High doses of aspirin will be continued for some time

The measles, mumps and rubella vaccine should be delayed Discharge instructions for a child with Kawasaki disease should include the information that immunoglobulin therapy may interfere with the body's ability to form appropriate amounts of antibodies. Therefore, live or attenuated (weakened) immunizations should be delayed. The MMR vaccine contains three live attenuated viruses and should be delayed until the child's immune system recovers from this treatment.

The nurse is caring for a toddler who is diagnosed with an infection and whose temperature is 103 F (39.4 C). Which intervention would be most effective in lowering the client's temperature? Immerse the child in a tub containing cool water Give a tepid sponge bath prior to giving an antipyretic Use medications to lower the temperature and improve comfort Apply extra layers of clothing to prevent shivering

Use medications to lower the temperature and improve comfort Fever is not the primary illness; it is a physiologic mechanism the body uses to fight an infection. The best response is to administer an antipyretic to normalize body temperature and improve the overall comfort of the child. Tepid sponge baths can lower the body temp, but they don't necessarily improve comfort, and you have to be careful not to chill the child because shivering would only increase the body temperature.

A woman in labor calls a nurse to assist her in the bathroom. The nurse notices a large amount of clear fluid on the bed linens. The nurse should act based on knowledge that fetal monitoring must now assess for what complication? Variable decelerations Early decelerations Periodic accelerations Late accelerations

Variable decelerations When the membranes rupture, there is increased risk initially of cord prolapse if the head is at a minus level. Fetal heart rate patterns may show variable decelerations, which require immediate nursing action to reposition the client, apply oxygen and notify the health care provider.

The nurse is preparing to administer a routine feeding through a gastrostomy tube (G-tube). Before administering the feeding, the nurse should conduct which priority nursing assessment? Verify G-tube patency Assess the breath sounds Palpate the abdomen Check the temperature

Verify G-tube patency G-tube patency should be checked prior to all feedings. If more than 125 mL of aspirate or residual is obtained, the feeding should be held. The residual should be returned into the stomach. The feeding should not be attempted if the tube is not patent or aspirate cannot be obtained.

The nurse is caring for a 4 year-old child with a greenstick fracture. In explaining this type of fracture to the parents, the best comment by the nurse should include which point? "Bone fragments often remain attached by a periosteal hinge." "A child's bone is more flexible and can be bent 45 degrees before breaking." "Compression of porous bones produces a buckle or torus type break." "Bones of children are more porous than adults' and often have incomplete breaks."

"Bones of children are more porous than adults' and often have incomplete breaks." This allows the pliable bones of growing children to bend, buckle, and break in a "greenstick" manner. A greenstick fracture occurs when a bone is angulated beyond the limits of bending. The compressed side bends and the tension side develops an incomplete fracture.

The mother of a 4 month-old infant asks the nurse about the dangers of sunburn while they are on vacation at the beach. Which of these statements is the best advice about sun protection for this child? "Liberally apply a sunscreen with a minimum sun protective "Sunscreen should not be used on children." "You should keep the baby inside unless it's cloudy outside." "Dress the infant in lightweight long pants, long-sleeved shirts and brimmed hats."

"Dress the infant in lightweight long pants, long-sleeved shirts and brimmed hats." Infants under 6 months of age should be kept out of the sun or shielded from it. Even on a cloudy day, the infant can be sunburned while near water. A hat and light protective clothing should be worn. Sunscreen is not generally recommended for infants under the age of 6 months; however, the American Academy of Pediatrics states that it can be applied to small areas of the baby's skin that are exposed to the sun (such as the baby's face or the back of the hands).

A client has just received an extracorporeal shock-wave lithotripsy (ESWL) procedure. What is the priority information the nurse should teach ? "Restrict milk and dairy products for one to two months." "Drink 3,000 to 4,000 mL of fluid each day for one month." "Limit fluid intake to 1,000 mL each day for two months." "Increase intake of citrus fruits to three servings per day for two months."

"Drink 3,000 to 4,000 mL of fluid each day for one month." Drinking three to four quarts (3,000 to 4,000 mL) of fluid each day will aid passage of fragments of the broken up renal calculi and help prevent formation of new calculi.

A client is started on long-term corticosteroid therapy. Which comment by the client indicates a need for more teaching? "I will be sure to eat foods high in potassium." "For one week every month I will stop taking the medication." "The medication needs to be taken with food." "I will keep a weekly weight record."

"For one week every month I will stop taking the medication." To suddenly stop taking a steroid may result in a sudden drop in the blood pressure from a loss in fluid volume associated with adrenal crisis. Clients should be warned not to abruptly stop taking the medication. Corticosteroids can lower the amount of potassium in the body so the client should eat more potassium-rich foods. Weight gain is an expected effect of corticosteroid therapy; clients should regularly keep track of their weight. Normally corticosteroid medications are taken with breakfast.

A client who lives in an assisted living facility tells the nurse, "I am so depressed. Life isn't worth living anymore." What is the best response by the nurse to this statement? "Come on, it is not that bad." "Have you thought about hurting yourself?" "Think of the many positive things in life today." "Did you tell any of this to your family?"

"Have you thought about hurting yourself?" It's important to determine if someone, who has voiced thoughts about death, is considering a suicidal act. This response is most therapeutic under the circumstances. To respond by saying things are not so bad, denies the validity of the client's statement. To ask if the family or anyone knows of these feelings lacks focus on the client and would also be in violation of the rights of the client.

The nurse is admitting a client who is newly diagnosed with a frontal lobe brain tumor. Which statement made by a spouse may provide important information about this diagnosis and should be communicated to the health care provider? "It seems our sex life is nonexistent over the past six months." "In the morning and evening he complains that reading is next to impossible because the print is blurry." "I find the mood swings and the change from being a calm person to being angry all the time hard to deal with." "His breathing rate is usually below 12."

"I find the mood swings and the change from being a calm person to being angry all the time hard to deal with." The frontal lobe of the brain controls affect, judgment and emotions. Dysfunction in this area results in findings such as emotional lability, changes in personality, inattentiveness, flat affect and inappropriate behavior.

At a senior citizen's group meeting the nurse talks with a client who has type 1 diabetes. Which statement by the client during the conversation is most predictive of a potential for impaired skin integrity? "I give my insulin to myself in my thighs and belly and alternate sites." "Here are my glucose test readings that I wrote on my calendar." "If I bathe more than once a week my skin feels too dry." "I had a penny in my shoe all day last week, and I didn't even realize it until I took my shoes off!"

"I had a penny in my shoe all day last week, and I didn't even realize it until I took my shoes off!" Peripheral neuropathy can lead to lack of sensation in the lower extremities. Clients who do not feel pressure and/or pain are at high risk for skin impairment.

A 15 year-old client has been placed in a Milwaukee brace. Which statement made by the client is incorrect and indicates a need for additional teaching? "I should inspect my skin under the brace every day" "I will only have to wear this for six months." "I can take it off when I shower or take a bath." "The brace has to be worn all day and night."

"I will only have to wear this for six months." The brace must be worn long-term, during periods of growth, usually for one to two years. It is used to correct scoliosis, the lateral curvature of the spine.

A client has had a positive reaction to purified protein derivative (PPD). When the client asks, "What does this mean?" the nurse should respond with which statement? "This means you have never had or been around someone with tuberculosis." "You have been exposed to the organism Mycobacterium tuberculosis." "You are mostly likely have a natural immunity to the bacteria." "You most likely have a resistant form of active tuberculosis."

"You have been exposed to the organism Mycobacterium tuberculosis." The PPD skin test is used to determine the presence of tuberculosis antibodies. In an otherwise healthy person, an induration greater than or equal to 15 mm is considered a positive skin test. This indicates that the client has been exposed to the organism Mycobacterium tuberculosis. Additional tests such as a chest x-ray and sputum culture will be needed to determine if active tuberculosis is present. The sputum cytology test is the only definitive test to confirm a diagnosis of active TB

A nurse entering the room of a postpartum mother observes the baby lying at the edge of the bed while the mother sits in a nearby chair. The mother states, "This is not my baby, and I do not want it." After repositioning the child safely, the nurse should respond with which comment? "This is a common occurrence after birth, but you will come to accept the baby." "Many women have postpartum blues and need some time to love the baby." "You seem upset. Tell me what the pregnancy and birth were like for you." "What a beautiful baby! Her eyes are just like yours and so is her smile."

"You seem upset. Tell me what the pregnancy and birth were like for you." A nonjudgmental, open ended response facilitates dialogue between the client and the nurse. The other three options ignore the situation and the needs of the mother. Note that the correct answer is the only client-centered option that is directly associated with the given situation.

A nurse is working with one licensed practical nurse (LPN) and a mental health tech (an unlicensed assistive personnel). Which newly admitted client would be appropriate to assign to the mental health tech? An adolescent diagnosed with dehydration and anorexia A young adult who reports to be a heroin addict and states, "I am in withdrawal and seeing spiders." A middle-aged client diagnosed with an obsessive compulsive disorder A 76 year-old client diagnosed with severe depression

A middle-aged client diagnosed with an obsessive compulsive disorder The mental health tech (a type of unlicensed assistive personnel or UAP) can be assigned to care for a client with a chronic condition after an initial assessment by the nurse. This client has minimal risk of instability of condition and has a situation of expected outcomes.

The clients listed below are all using patient-controlled analgesic (PCA) pump for pain control. Which of these clients is least appropriate to use a PCA pump? A preschooler with intermittent episodes of alertness An older adult client with numerous arthritic nodules on the hands A teenager who reads at a 4th-grade level A young adult with a history of Down syndrome

A preschooler with intermittent episodes of alertness A preschooler is the one client most likely to have difficulty with the use or understanding of a PCA pump. The preschooler also has a decreased level of consciousness and would not be able to fully benefit from the use of a PCA pump. School-age children, ages 6 and up, are better candidates for PCA electronic pumps.

A client expresses anger when a call light is not answered within five minutes. The client demanded a blanket. How should the nurse respond? A. "I see this is frustrating for you. I have a few minutes so let's talk." B. "I apologize for the delay. I was involved in an emergency." C. "Let's talk. Why are you upset about this?" D. "I am surprised that you are upset. The request could have waited a few more minutes."

A. "I see this is frustrating for you. I have a few minutes so let's talk." This is the best response because it gives credence to the client's feelings and then concerns. To say "let's talk" and ask a why question is not a therapeutic approach because it does not acknowledge or validate the client's feelings. To apologize and not notice the client's feelings is inappropriate. To say it could have waited a few minutes is rude and non-accepting of the client's verbalized needs

The nurse is examining a 2 year-old child with a tentative diagnosis of Wilm's tumor. The nurse would be most concerned about which statement by the mother? A. "Urinary output seems to be less over the past two days." B. "My child has lost three pounds in the last month." C. "The child prefers some salty foods more than others." D. "All the pants have become tight around the waist."

A. "Urinary output seems to be less over the past two days." Wilm's tumor is a malignant tumor of the kidney that can lead to kidney dysfunction; therefore, a recent decrease in urinary output should be investigated further as it may be a sign of renal dysfunction. Increasing abdominal girth is a common finding in Wilm's tumor, but does not require immediate intervention by the nurse.

Which individual is at greatest risk for the development of hypertension? A. 45 year-old African-American attorney B. 40 year-old Caucasian nurse C. 60 year-old Asian-American shop owner D. 55 year-old Hispanic teacher

A. 45 year-old African-American attorney The incidence of hypertension is greater among African-Americans than other groups in the United States. The incidence among the Hispanic population is rising.

The nurse receives a client from the post anesthesia care unit following a left femoral-popliteal bypass graft procedure. Which of the following assessments requires immediate notification of the health care provider? A. Absent left pedal pulse using Doppler analysis B. Acute pain in the left lower leg C.Left foot is cool to the touch D.Inability to palpate the left pedal pulse

A. Absent left pedal pulse using Doppler analysiS Although the inability to palpate the left pedal pulse, a cool extremity, and increased pain in the left lower leg are important findings, they all require additional nursing assessment prior to contacting the health care provider. In clients without palpable pedal pulses, the next step in the assessment is to perform a Doppler analysis. The inability to locate the left pedal pulse using the Doppler analysis requires immediately notifying the health care provider.

A woman, who delivered five days ago and who had been diagnosed with pregnancy induced hypertension (PIH), calls a hospital triage nurse hotline to ask for advice. She states, "I have had the worst headache for the past two days. It pounds and by the middle of the afternoon everything I look at looks wavy. Nothing I have taken helps." What should the nurse do next? A. Ask the client to stay on the line, get the address, and send an ambulance to the home B. Advise the client that the swings in her hormones may be the problem; suggest that she call her health care provider C. Advise the client to have someone bring her to the emergency room as soon as possible D. Ask the client to explain what she has taken and how often, and then evaluate other specific complaints

A. Ask the client to stay on the line, get the address, and send an ambulance to the home The woman is at risk for seizure activity. The ambulance needs to bring the woman to the hospital for evaluation and treatment. For at-risk clients, PIH may progress to preeclampsia and eclampsia prior to, during, or after delivery; this may occur up to 10 days after delivery.

The nurse is reviewing the laboratory results for several clients. Which laboratory result indicates that one of the clients is in metabolic acidosis? A. Carbon dioxide 20 mEq/L B. Chloride 100 mEq/L C. Hemoglobin 15 gm/dL D. Sodium 130 mEq/L

A. Carbon dioxide 20 mEq/L Serum carbon dioxide is an indicator of acid-base status. This finding would indicate metabolic acidosis because the lungs compensate by blowing off CO2 so it will be low. Normal carbon dioxide is 35 to 45 mEq/L.

The nurse is caring for a client who is in the advanced stage of multiple myeloma. Which action should be included in the plan of care? A. Careful repositioning B. Monitor for hyperkalemia C. Place in protective isolation D. Administer diuretics as ordered

A. Careful repositioning Multiple myeloma occurs when abnormal plasma cells (myeloma cells) collect in several bones. This disease may also harm other tissues and organs, especially the kidneys. This type of cancer causes hypercalcemia, renal failure, anemia,and bone damage. Because multiple myeloma can cause erosion of bone mass and fractures, extra care should be taken when moving or positioning a client due to the risk of pathological fractures.

The client with a T-2 spinal cord injury reports having a "pounding" headache. Further assessment by the nurse reveals excessive sweating, rash, pilomotor erection, facial flushing, congested nasal passages and a heart rate of 50. What action should the nurse take next? A. Check the client for bladder distention and the urinary catheter for kinks B. Measure the client's respirations, blood pressure, temperature and pupillary responses C. Place the client into the bed and administer the ordered PRN analgesic D. Assist client with relaxation techniques

A. Check the client for bladder distention and the urinary catheter for kinks These are findings of autonomic dysreflexia, also called hyperreflexia. This response occurs in clients with a spinal cord injury above the T-6 level. It is typically initiated by any noxious stimulus below the level of injury such as a full bladder, an enema or bowel movement, fecal impaction, uterine contractions, changing of the catheter and vaginal or rectal examinations. The stimulus creates an exaggerated response of the sympathetic nervous system and can be a life-threatening event. The BP is typically extremely high. The priority action of the nurse is to identify and relieve the cause of the stimulus.

The clinic nurse is assisting with medical billing. The nurse uses the DRG (Diagnosis Related Group) manual for which purpose? A. Determine reimbursement for a medical diagnosis B. Implement nursing care based on case management protocol C. Identify findings related to a medical diagnosis D. Classify nursing diagnoses from the client's health history

A. Determine reimbursement for a medical diagnosis DRGs are the basis of prospective payment plans for reimbursement for Medicare clients. Other insurance companies often use it as a standard for determining payment.

The client is prescribed dexamethasone by mouth every other day and asks the nurse for more information about the medication. What information would the nurse want to share with the client? (Select all that apply) A. Do not get any immunizations or skin tests B. Avoid dairy products C. Take the medication with food D. Take the medication at bedtime E. Mark your calendar to keep track of doses

A. Do not get any immunizations or skin tests C. Take the medication with food E. Mark your calendar to keep track of doses Dexamethasone is a glucocorticosteroid used for its anti-inflammatory properties. It is best to take the medication in the morning, before 9:00 am, with food or milk to avoid stomach upset. A low-sodium diet is usually prescribed because the drug can cause an elevation in blood pressure, salt and water retention, and increased potassium loss. Dexamethasone also causes calcium loss; the client should increase calcium in the diet and take a calcium supplement. Because the medication affects the immune system, it could make vaccinations ineffective and/or lead to serious infections. It's always a good idea for clients to keep track of medication administration, particularly when they are not taking the medication every day.

A nurse is planning care for a 2 year-old hospitalized child. Which issue will produce the most stress at this age? A. Separation anxiety B. Loss of control C. Bodily injury D. Fear of pain

A. Separation anxiety While a toddler will experience all of the stresses, separation from parents is the major stressor. Separation anxiety peaks in the toddler years.

The nurse working in the intensive care unit (ICU) is told that a client is being newly admitted with a diagnosis of hyperglycemic hyperosmolar nonketotic state (HHNS). The nurse would expect which of the following clinical findings in this client? (Select all that apply.) A. Severe dehydration B. Ketonuria C. Blood glucose level of at least 600 mg/dL D. Metabolic acidosis E. History of type 1 diabetes mellitus

A. Severe dehydration C. Blood glucose level of at least 600 mg/dL The typical client with HHNS will have a plasma glucose level of 600 mg/dL or greater, high serum osmolality, profound dehydration, a serum pH greater than 7.3 and some alteration in consciousness. Unlike diabetic ketoacidosis, however, there is little to no ketosis. HHNS usually presents in older clients with type 2 diabetes mellitus who have some concomitant illness (usually an infection) that leads to reduced fluid intake, or who do not adhere to their diabetic medications and diet. All clients with HHNS require hospitalization and rapid treatment to correct the profound hypovolemia and hyperglycemia characteristic of this condition.

A nurse is working in an OB-GYN clinic. A 40 year-old woman in the first trimester of an unplanned pregnancy provides a health history to the nurse. Which information should receive priority attention? A. She has been taking an ACE inhibitor for her blood pressure for the past two years. B. Her husband was treated for tuberculosis as a child C. Her father and brother have type 1 diabetes D. She has taken 800 mcg of folic acid daily for the past year

A. She has been taking an ACE inhibitor for her blood pressure for the past two years. A report by the client that she has been taking medications in the first trimester of pregnancy should be followed up immediately. ACE inhibitors, commonly used to control high blood pressure, are pregnancy category X, as they can cause teratogenic effects on the developing fetus, increasing the risk of birth defects. Women who are taking medications and who are planning a pregnancy should be switched to medications that are not harmful to the developing fetus before they begin trying to get pregnant.

A newborn is diagnosed with hypothyroidism. In discussing the condition and treatment with the family, the nurse should emphasize which point? This rare problem is always hereditary They can expect the child will be mentally retarded Physical growth and development will be delayed Administration of thyroid hormone will prevent complications

Administration of thyroid hormone will prevent complications Early identification (ideally before 13 days-old) and continued treatment with levothyroxine (thyroid hormone replacement) corrects hypothyroidism in newborns, preventing problems. If undetected and untreated, hypothyroidism can result in poor growth and weight gain, slow heart rate, low blood pressure, and babies who are unusually quiet; the child will be at risk for permanent brain damage and intellectual disabilities. Approximately one in every 4000 babies is born with hypothyroidism.

At the beginning of the shift, the nurse is reviewing the status of each of the assigned clients in the labor and delivery unit. Which of these clients should the nurse check first? An adolescent who is 18-weeks pregnant with a report of no fetal heart tones and is coughing up frothy sputum A young woman who is a grand multipara, cervical dilation to 4 cm and is 50% effaced A middle-aged woman with a history of two prior vaginal term births and who is 2 cm dilated A young woman, first-time para, cervical dilation to 1 cm and contractions 15 minutes apart

An adolescent who is 18-weeks pregnant with a report of no fetal heart tones and is coughing up frothy sputum The 18 year-old client has an actual complication of left-sided heart failure and a possible stillborn birth. The other clients present with findings of potential, but not actual, complications.

The charge nurse is scheduled duties in the unit. Which of these clients should a charge nurse assign to a licensed practical nurse (LPN)? A trauma victim with multiple lacerations that require complex dressings An older adult client diagnosed with cystitis and an indwelling urethral catheter A confused client whose family complains about the nursing care two days after the client's surgery A client admitted with the diagnosis of possible transient ischemic attack with unstable neurological signs

An older adult client diagnosed with cystitis and an indwelling urethral catheter This is the most stable client who has more chance of predictable outcomes and minimal risk for complications. The other clients require more complex care and would need the attention of the RN.

Following an alert of an internal disaster and the need for beds, the charge nurse is asked to list the clients who are potential discharges within the next hour. Which one of these clients should the charge nurse select? A middle-aged client who was admitted yesterday with an internal automatic defibrillator and complaints of "passing out at unknown times" An older adult client who has been diagnosed with type 2 diabetes mellitus for more than 20 years, admitted with diabetic ketoacidosis 24 hours ago An adolescent admitted the prior night with Tylenol intoxication A school-aged child who was admitted at the change of shifts with a diagnosis of suspected bacterial meningitis

An older adult client who has been diagnosed with type 2 diabetes mellitus for more than 20 years, admitted with diabetic ketoacidosis 24 hours ago This client is the most stable and has a chronic condition. Tylenol intoxication requires at least three to four days of intensive observation for the risk of hepatic failure. The other clients should be considered unstable.

The nurse is preparing to administer albuterol inhaled (AccuNeb, ProAir HFA, Proventil HFA, Ventolin HFA) to a 11 year-old with asthma. Which assessment by the nurse indicates there is a need for the health care provider to adjust the medication? Apical pulse of 112 Lower extremity edema Lethargy Temperature of 101 F (38.3 C)

Apical pulse of 112 One of the more common adverse effects of beta adrenergic medications is an increase in heart rate. Normal resting heart rate for children 10 years and older is the same as adults: 60-100 beats per minute.

A 3 year-old has just returned from surgery for application of a hip spica cast. What nursing action will be the priority? Apply waterproof plastic tape to the cast around the genital area Position the child flat in bed, repositioning from back to stomach every two to four hours Drying the cast using a hair dryer set to "warm" Use the crossbar to help turn the child from side to side

Apply waterproof plastic tape to the cast around the genital area The most important aspects of caring for the cast is to keep it clean and dry. Shortly after returning from surgery, waterproof plastic tape will be applied around the genital area to prevent soiling. The child should be turned every two hours to help facilitate drying, from side to side and front to back, with the head elevated at all times. If a crossbar is used to stabilize the legs, it should not be used to turn the child (it may break off). After the cast has completely dried and it becomes damp, it can be either exposed to air or a hair dryer (set to warm or cool) may be used to help dry the cast.

The nurse, who is participating in a community health fair, assesses the health status of attendees. When would the nurse conduct a mental status examination? As part of every health assessment The individual displays restlessness The individual reports memory lapses There are obvious signs of depression

As part of every health assessment A mental status assessment is a critical part of baseline information and should be a part of every examination.

n 88 year-old client is admitted to the telemetry unit following a minor surgical procedure. The client's history includes insulin dependent diabetes and a previous myocardial infarction. The nurse responds to the client's ECG alarm and finds the client's rhythm shows asystole and the client obtunded but responsive. Prioritize the actions of the nurse (with 1 being the top priority). Assess respirations and pulse Check a blood glucose level Look at a different ECG lead to confirm rhythm Initiate emergency response system if indicated

Assess respirations and pulse Initiate emergency response system if indicated Look at a different ECG lead to confirm rhythm Check a blood glucose level After checking responsiveness, establishing a patent airway and then assessing breathing and circulation are the next priorities (ABCs). This assessment would provide information to decide whether the emergency response team is needed. Because the client is responsive, the monitor rhythm is not correct, as a client with asystole would be unresponsive. Asystole on a rhythm strip may simply be a loose lead; a quick way to check this is to select another lead. The client's obtunded state indicates that ion is needed, so assessment of a central pulse and blood pressure is indicated to determine whether cardiovascular compromise is responsible for this condition. If no evidence of an immediate cardiac event is present, the blood glucose should be checked. Stress and changes in food or fluid consumption secondary to surgery increase the risk of glucose imbalance in the person with diabetes.

A nurse is caring for a client diagnosed with chronic obstructive pulmonary disease (COPD) and who becomes dyspneic. The nurse should take which action? Instruct the client to breathe into a paper bag Assist the client with pursed-lip breathing Place the client in a low Fowler's position Administer oxygen at six liters per minute via nasal cannula

Assist the client with pursed-lip breathing Pursed-lip breathing should be encouraged during periods of dyspnea in COPD to control rate and depth of respiration, to prevent alveolar collapse and to improve respiratory muscle coordination. Clients with COPD are usually on lower doses of oxygen, titrated to maintain an oxygen saturation of 88-91%. Semi-Fowler's position is usually most comfortable for someone with COPD, because this position allows the client's diaphragm to expand.

A client who is two days postop, has these vital signs: blood pressure of 120/70, heart rate of 110 BPM, respiratory rate of 26, and a temperature of 100.4 F (38 C). The client suddenly becomes profoundly short of breath (SOB) and the skin color becomes grayish in color. Which assessment should the a nurse do first based on the client's change in condition? Check for orthostatic hypotension Palpate the pulses for bounding and irregularity Auscultate for diminished breath sounds Assess the pupils for unequal responses to light

Auscultate for diminished breath sounds The findings suggest pulmonary embolus as a result of a piece of a clot in the legs that has broken off. Thus, the breath sound will most likely be diminished or absent in the lung where the embolus lodged.

The nurse manager informs the nursing staff that the clinical nurse specialist will be conducting a research study on staff attitudes toward client care, and all staff are invited to participate in the study if they wish. This affirms which ethical principle? Anonymity Beneficence Autonomy Justice

Autonomy Individuals must be free to make independent decisions about participation in research without coercion from others. Anonymity means the person's identity is not revealed. Beneficence is the state or quality of being kind, charitable, beneficial or a charitable act.

The client is diagnosed with gastroesophageal reflux disease (GERD). Which recommendation made by the nurse would be most helpful? Avoid liquids unless a thickening agent is used Sit upright for at least half an hour after eating Avoid eating two hours before going to sleep Maintain a diet of soft foods and cooked vegetables

Avoid eating two hours before going to sleep Eating before sleeping enhances regurgitation of stomach contents, which have increased acidity, into the esophagus. An upright posture should be maintained for about two hours after eating to allow for stomach emptying. Thickening agents and sitting upright for an hour after eating are interventions for clients with swallowing difficulties. Common dietary modifications in GERD include decreasing total fat intake; avoiding large meals, chocolate, caffeine or known irritants; a soft diet is not necessary.

A 67 year-old client is admitted with substernal chest pressure that radiates to the jaw. The admitting diagnosis is acute myocardial infarction (MI). What should be the priority nursing diagnosis for this client during the first 24 hours? A. Anxiety B. Altered tissue perfusion C. Activity intolerance D. Risk for fluid volume excess

B. Altered tissue perfusion In the immediate post MI period, altered tissue perfusion is priority, as an area of myocardial tissue has been damaged by a lack of blood flow and oxygenation. Interventions should be directed toward promoting tissue perfusion and oxygenation. The other problems are also relevant, but tissue perfusion is the priority.

A nurse is caring for a client two hours after a right lower lobectomy. The chest drainage system is connected to -20 cm suction. During the assessment, the nurse notes that there is constant bubbling in the water seal chamber. What is the first action the nurse should take? A. Reposition the client to improve the level of comfort B. Assess the chest tube dressing, tubing and drainage system C. Check for any increase in the amount of thoracic drainage D. Call the health care provider as soon as possible

B. Assess the chest tube dressing, tubing and drainage system Continuous bubbling in the water seal chamber indicates an air leak. The nurse should first assess the dressing and tubing for an air leak in the system. If there is no leak, then there is an air leak in the lung. In the immediate postoperative period, this is a common finding that should be documented and reassessed periodically. As the lung heals after surgery, the air leak should resolve. Persistence of the air leak after seven days is abnormal and may require further intervention. A new air leak that was not present on previous assessment would also be a significant finding that would require notification of the provider with possible intervention.

The nurse is teaching a client with coronary artery disease about nutrition. What information should the nurse be sure to emphasize? A. Add complex carbohydrates to each meal B. Avoid large and heavy meals C. Limit sodium to 7 grams per day D. Eat three balanced meals a day

B. Avoid large and heavy meals Eating large, heavy meals can pull blood away from the heart for the digestion process. This may result in angina for clients with coronary artery disease. Sodium for clients with cardiac disease is limited to two grams per day. Three meals a day is a correct approach. However, it does not mention the size of the meal, which is more important.

A child is treated with succimer (Chemet) for lead poisoning. Which of these assessments should the nurse perform first? A. Test deep tendon reflexes B. Check complete blood count (CBC) with differential C. Check serum potassium level D. Check blood calcium level

B. Check complete blood count (CBC) with differential Succimer (Chemet) is used in the management of lead or other heavy metal poisoning. Although it has generally well tolerated and has a relatively low toxicity, it may cause neutropenia. Therapy should be withheld or discontinued if the absolute neutrophil count (ANC) is below 1200/µ.

A client has a chest tube inserted immediately after surgery for a left lower lobectomy. During the repositioning of the client during the first postop check, the nurse notices 75 mL of a dark, red fluid flowing into the collection chamber of the chest drain system. What is the appropriate nursing action? A. Call the surgeon immediately B. Continue to monitor the rate of drainage C. Turn the client back to the original position D. Check to see if the client has a type and cross match

B. Continue to monitor the rate of drainage It is not unusual for blood to collect in the chest and be released into the chest drain when the client changes position this soon after surgery. The dark color of the blood indicates it is not active bleeding inside of the chest. Sanguinous drainage should be expected within the initial 24 hours postop, progressing to serosanguinous and then to a serous type. If the drainage exceeds 100 mL/hr, the nurse should call the surgeon.

The nurse is caring for a client diagnosed with cystic fibrosis. The nurse would expect this client to receive supplemental pancreatic enzymes along with a diet with what characteristics? A. High in carbohydrates, low in proteins B. High in carbohydrates and proteins C. Low in carbohydrates, high in proteins D. Low in carbohydrates and proteins

B. High in carbohydrates and proteins A high-energy diet is needed because of the efforts needed to expectorate secretions. Thus, an increase in calories is achieved with high carbohydrates, protein and fat (possibly as high as 40%). A favorable response to the supplemental pancreatic enzymes is based on tolerance of fatty foods, decreased stool frequency, absence of steatorrhea, improved appetite and lack of abdominal pain. Salt must also be supplemented because excessive salt is lost through sweat.

There is an order to administer intravenous gentamicin three times a day. What diagnostic finding indicates the client may be more likely to experience a toxic side effect of this medication? A. Low serum blood urea nitrogen B. High serum creatinine C. High gastric pH D. Low serum albumin

B. High serum creatinine Gentamicin is excreted unmodified by the kidneys. If there is any reduced renal function, toxicity can result. An elevated serum creatinine indicates reduced renal function and this puts the client at greater risk for toxicity. Reduced renal function will delay the excretion of many medications.

A client returns from the operating room after a right orchiectomy. Which of the outcomes below would be the priority for the nurse postoperatively? A. Ambulate the client within a few hours after surgery B. Manage postoperative pain C. Maintain fluid and electrolyte balance D. Control bladder spasms

B. Manage postoperative pain Orchiectomy is the removal of the testicles; the penis and the scrotum are left intact. It is usually done to stop production of testosterone; this helps relieve symptoms, prevents complications and prolongs survival for advanced prostate caner. Due to the location of the incision, pain management is the priority. Bladder spasms are more related to prostate surgery. However, the other options may be a later focus.

The nurse is assessing a 4 year-old child who is in skeletal traction 24 hours after surgical repair of a fractured femur. The child is crying and reports having severe pain. The right foot is pale and there is no palpable pulse. What action should the nurse take first? A. Reassess the extremity in 15 minutes B. Notify the health care provider C. Administer the ordered PRN medication D. Readjust the traction for comfort

B. Notify the health care provider Pain and absence of a pulse within 48-72 hours after a severe injury to an extremity suggests acute compartment syndrome. This condition occurs when there's a build up of pressure within the muscles; this pressure decreases blood flow and can cause muscle and nerve damage. Acute compartment syndrome is a medical emergency. Surgery is needed immediately; delaying surgery can lead to permanent damage to the extremity.

A nurse is teaching a group of adults about modifiable cardiac risk factors. Which of the following should the nurse focus on first? A. Weight reduction B. Smoking cessation C. Physical exercise D. Stress management

B. Smoking cessation Smoking cessation is the priority for clients at risk for cardiac disease. Smoking's effects result in reduction of cell oxygenation and constriction of the blood vessels. All of the other factors should be addressed at some point in time.

A client, admitted to the unit because of severe depression and suicidal threats, is placed on suicidal precautions. The nurse should be aware that the danger of the client committing suicide is greatest at what period of time? A. At the time of the client's greatest despair B. When the client's mood improves with an increase in energy level C. After a visit from the client's estranged partner D. During the night shift when staffing is limited

B. When the client's mood improves with an increase in energy level Suicide potential is often increased when there is an improvement in mood and energy level. At this time ambivalence is often decreased and a decision is made to commit suicide. The clients have the energy to carry through with the plan for suicide.

A nurse is teaching a client to select foods rich in potassium to prevent digitalis toxicity. Which choice indicates the client understands this dietary requirement and recognizes which foods are highest in potassium? Baked potato Small banana Three apricots Naval orange

Baked potato A baked potato contains 610 milligrams of potassium. Apricots, oranges and bananas do have higher potassium content, but because of their size they are not the highest in potassium. A baked potato is the highest in potassium of the given options.

Two members of the interdisciplinary team are arguing about the plan of care for a client. Which action could any one of the members of the team use as a de-escalation strategy? Bring the communication focus back to the client Interrupt, apologize for interruption, and change the subject Tell the violators they must calm down and be reasonable Adjourn the meeting and reschedule when everyone has calmed down

Bring the communication focus back to the client Bringing the subject of the communication back to the client refocuses attention on the client's care, instead of the manner of communication. It is the most effective strategy because it is an example of collaboration. The other options are non-productive and may even make matters worse.

The nurse is providing information to a client about a prescribed medication. Which one of these statements, if made by a client, indicates that teaching about propranolol (Inderal) has been effective? A. "I could have an increase in my heart rate for a few weeks." B. "I will expect to feel nervousness the first few weeks." C. "I can have a heart attack if I stop this medication suddenly." D. "I may experience seizures if I stop the medication abruptly."

C. "I can have a heart attack if I stop this medication suddenly." Propranolol is commonly used to treat hypertension, abnormal heart rhythms, heart disease and certain types of tremor. It is in a class of medications called beta blockers. Suddenly discontinuing a beta blocker can cause angina, hypertension, arrhythmias, or even a heart attack.

The nurse is developing a teaching plan for parents on safety and risk-reduction in the home. Which of the following should the nurse give priority consideration to during teaching? A. Proximity to emergency services B. Number of children in the home C. Age of children in the home D. Age and knowledge level of the parents

C. Age of children in the home Age and developmental level of the child are the most important considerations in the provision of a framework for anticipatory guidance associated with safety, and should be given priority when teaching safety.

The client is scheduled for coronary artery bypass. Based on principles of teaching and learning, what is the best initial approach by the nurse during pre-op teaching? A. Administer a written pretest B. Mail a videotape to the home C. Assess the client's learning style D. Tour the coronary intensive unit

C. Assess the client's learning style As with any anticipatory teaching, assessment of the client's level of knowledge and learning style should occur first. If possible, the three senses of hearing, seeing and touching should be used during any teaching to enhance recall.

The nurse is caring for a client diagnosed with acute angina. The client is receiving an intravenous infusion of nitroglycerin. What is the priority assessment during this treatment? A. Heart rate B. Neurologic status C. Blood pressure D. Urine output

C. Blood pressure The vasodilatation that occurs as a result of this medication can cause profound hypotension. The client's blood pressure must be evaluated every 15 minutes until stable, and then every 30 minutes to every hour thereafter. Clients receiving IV nitroglycerin also require continuous ECG monitoring.

A client with a diagnosis of methicillin-resistant Staphylococcus aureus (MRSA) has died. Which type of precautions is appropriate to use when performing postmortem care? A. Compromised host precautions B. Airborne precautions C. Contact precautions D. Droplet precautions

C. Contact precautions The resistant bacteria remain alive for up to three days after the client dies. Therefore, contact precautions must still be used. The body should also be labeled as MRSA-contaminated so that the funeral home staff can protect themselves as well. Gown and gloves are required.

A client who is receiving chemotherapy through a central line is admitted to the hospital with a diagnosis of sepsis. Which nursing intervention should receive priority? A. Change the dressing over the site of the central line B. Place the client in reverse isolation C. Inspect all sites that may serve as entry ports for bacteria D. Restrict contact with persons having known, or recent, infections

C. Inspect all sites that may serve as entry ports for bacteria Prompt recognition of source of infection and subsequent initiation of therapy will reduce morbidity and mortality.

During assessment of orthostatic vital signs on a client with cardiomyopathy, the nurse finds that the systolic blood pressure (BP) decreased from 145 to 110 mm Hg between the supine and upright positions while the heart rate (HR) rose from 72 to 96 beats per minute. In addition, the client reports feeling lightheaded when standing up. The nurse should implement which of the following actions? A. Restrict fluids for the next few hours B. Increase fluids that are high in protein C. Instruct the client to increase fluid intake for the next two days D. Instruct client to increase fluid intake for several hours

C. Instruct the client to increase fluid intake for the next two days This client is experiencing postural hypotension, a decrease in systolic blood pressure 15 mm Hg accompanied by an increase in heart rate 15 to 20 beats above the baseline with a change in position from supine to upright. This is often accompanied by lightheadedness. Fluid replacement is appropriate, but must be instituted very cautiously, as this client with cardiomyopathy will also be very sensitive to changes in fluid status and fluid overload may develop rapidly with aggressive rehydration. After the client increases fluid intake for one to two hours, the client should be reassessed for resolution of the postural hypotension.

The nurse is teaching the client with chronic renal failure (CRF) about medications. The client questions the purpose of taking aluminum hydroxide (Alternagel, Amphojel). What is the best explanation for the nurse to give the client about the therapeutic effects of this medication? A. The drug is taken to control gastric acid secretion B. It will reduce serum calcium C. It decreases serum phosphate D. Amphojel increases urine output

C. It decreases serum phosphate Aluminum binds phosphates that tend to accumulate in the client with chronic renal failure due to decreased filtration capacity of the kidney. Antacids such as Amphojel are commonly used to decrease serum phosphate.

Today's prothrombin time for a client receiving warfarin (Coumadin, Jantoven) is 20 seconds. The normal range listed by the lab is 10 to 14 seconds. What is an appropriate nursing action? A. Assess for bleeding gums or IV sites B. Observe the client for hematoma development C. Recognize that this is a therapeutic level D. Notify the health care provider immediately

C. Recognize that this is a therapeutic level For the client on warfarin therapy, this prothrombin level is within the therapeutic range. Therapeutic levels for warfarin are usually 1 1/2 to 2 times the normal levels.

The client with cancer is being treated with a biological response modifier. Which of the following side effects does the nurse anticipate with biologic therapy? Photophobia and sun sensitivity Hematuria Chills and fever Constipation

Chills and fever

A nurse is teaching adolescents about sexually transmitted diseases. What should the nurse emphasize is the most common infection? Human immunodeficiency virus (HIV) Gonorrhea Herpes Chlamydia

Chlamydia Chlamydia is the most frequently reported bacterial sexually transmitted disease in the United States. Prevention is similar to safe sex practices taught to prevent any sexually transmitted disease, such as abstinence, and the use of a condom and spermicide for protection during intercourse. This infection has subtle findings so the infected persons are less likely to pursue medical attention

The nurse is caring for a client who is experiencing a hypertensive crisis. The priority assessment in the first hour of care after admission to the critical care unit should focus on which factor? Heart rate Cognitive function Pedal pulses Lung sounds

Cognitive function The organ most susceptible to damage in hypertensive crisis is the brain, due to rupture of the cerebral blood vessels. Neurologic findings must be closely monitored.

The nurse is caring for a child diagnosed with Kawasaki disease (mucocutaneous lymph node syndrome or infantile polyarteritis). The nursing care plan should be based on the knowledge that this child is at risk for developing what complication? Coronary artery aneurysm Chronic vessel plaque formation Pulmonary embolism Occlusions at the vessel bifurcations

Coronary artery aneurysm Kawasaki disease affects the mucus membranes, lymph nodes, walls of the blood vessels and the heart. It can cause inflammation of the arteries, especially the coronary arteries of the heart, which can lead to aneurysms and possible heart attack in the child.

An external disaster has occurred in the town. The triage nurse from the emergency department is transported to the site and assigned to triage the injured. Which of these clients would the nurse tag as "to be seen last" by the providers at the scene? A. An older adult person with a open fracture of the left arm B. A teenager with small amount of bright red blood dripping out of the nose C. An infant with bilateral fractured lower legs with no active bleeding D. A middle-aged person with deep abrasions that are over 90% of the body

D. A middle-aged person with deep abrasions that are over 90% of the body The clients that are least likely to survive are to be tagged as the "last to be seen." Deep abrasions are usually treated as second or third degree burns because the fluid loss is great.

The RN is planning the care of an 80 year-old client with skin abrasions from a fall in the home. What aspect of this client's care is the primary responsibility of the nurse? A. Identification of a change in skin color B. Apply lotion to unaffected areas C. Report the finding of any break in the skin D. Assessment of the integumentary condition

D. Assessment of the integumentary condition The RN is ultimately responsible for thorough, ongoing assessment and evaluation of integument for this client. Because the nurse is responsible for all care-related decisions, only implementation tasks that do not require independent judgment can be delegated

The nurse needs to accurately assess gastric placement of a nasogastric tube prior to the administration of an enteral feeding. What is the priority action the nurse should take before starting the infusion? A. Place the end of the tube in water to check for air bubbles B. Measure the length of tubing from nose to epigastrium C. Auscultate the abdomen while instilling 10 mL of air into the tube D. Check the pH of the aspirate

D. Check the pH of the aspirate Once the initial placement of the tube has been confirmed by x-ray, the nurse will check the pH of the aspirate before administering medications or enteral feeding solutions. Current practice recommendations include assessing the feeding tube placement by testing the pH of aspirates, measuring the external portion of the tube, and observing for changes in the volume and appearance of feeding tube aspirates. If tube placement is in doubt, an x-ray should be obtained. The other methods are older approaches that are no longer recommended.

A nurse is providing care to a primigravida whose membranes spontaneously ruptured (ROM) four hours ago. At the time of rupture, maternal vital signs were within normal limits, she was dilated to 2 centimeters, and the baseline fetal heart rate (FHR) was 150 beats per minute (BPM). The nurse is now reassessing the client. Which of these assessment findings may be an early indication that the client is developing a complication of the labor process? A. Maternal temperature is 100 F (37.7 C) B. Cervical dilation of 4 centimeters C. Blood pressure is 138/88 mm Hg D. Fetal heart rate is 188 beats/minute

D. Fetal heart rate is 188 beats/minute olonged ruptured membranes may lead to maternal infection (as suggested by the slightly elevated temperature). But the primary concern is the fetal heart rate of 188; fetal heart rate is typically somewhere between 120 and 160 BPM. Fetal tachycardia may be an early sign of hypoxia. The nurse should contact the health care provider, assist the client to change positions, and administer oxygen and intravenous fluids.

A mother asks about expected motor skill development for her 3 year-old child. Which activity is considered a typical motor skill for the 3 year-old? A. Jumping rope B. Tying shoelaces C. Playing hopscotch D. Riding a tricycle

D. Riding a tricycle Three year-old children are developing gross motor skills that require large muscle movement. While there will always be some variation between children, movement milestones typically include pedaling a tricycle, standing on one foot for a few seconds, walking backwards and jumping with both feet. The other activities listed require more coordination and are movement milestones for older children.

The nurse is caring for a client in the late stages of amyotrophic lateral sclerosis (ALS). Which finding would the nurse expect? A. Tonic-clonic seizures B. Loss of half of visual field C. Confusion D. Shallow respirations

D. Shallow respirations ALS is a chronic progressive disease that involves degeneration of the anterior horn of the spinal cord as well as the corticospinal tracts. When the intercostal muscles and diaphragm become involved, the respirations become shallow and coughing is ineffective. The other findings are not typically expected with this diagnosis.

A client is transported to the emergency department after a motor vehicle accident. When assessing the client 30 minutes after admission, the nurse notes several physical changes. Which finding would require the nurse's immediate attention? A. Tachypnea B. Increased restlessness C. Tachycardia D. Tracheal deviation

D. Tracheal deviation Tracheal deviation is a sign that a mediastinal shift has occurred, most likely due to a tension pneumothorax. Air escaping from the injured lung into the pleural cavity causes pressure to build, collapsing the lung and shifting the mediastinum to the opposite side. This obstructs venous return to the heart, leading to circulatory instability and may result in cardiac arrest. This is a medical emergency, requiring emergency placement of a chest tube to remove air from the pleural cavity relieving the pressure.

The nurse is providing discharge teaching to a client who has had a total hip prosthesis implanted. During teaching, the nurse should include which content in the instructions for home care? Ambulate using crutches only Do not cross your legs at the ankles or knees Sleep only on your back and not on your side Avoid climbing stairs for three months

Do not cross your legs at the ankles or knees These clients should avoid the bringing of the knees together. Clients are to use a pillow between their legs when lying down and can lie on the back or side. Crossing the legs or bringing the knees together results in a strain on the hip joint. This increases the risk of a malfunction of the prosthesis where the ball may pop out. A walker or crutches may be used as assistive devices. These and other precautions are minimally followed for six weeks postoperative and sometimes longer as indicated.

The nurse is instructing a 65 year-old female client diagnosed with osteoporosis. What is the most important instruction about exercise? Exercise to reduce weight over a few months Avoid exercise activities that increase the risk of fracture Do weight-bearing or resistance activities Use exercise to strengthen muscles and protect bones

Do weight-bearing or resistance activities Weight-bearing or resistance exercises are beneficial in the treatment of osteoporosis. Although loss of bone cannot be substantially reversed, further loss can be greatly reduced if the client includes these exercises. In addition, other approaches are estrogen replacement and calcium supplements in a treatment protocol.

A nurse is teaching a class on human immunodeficiency virus (HIV) prevention. Which activity should be cautioned against since it is shown to increase the risk of HIV? Engaging in unprotected sexual encounters Physical touch of a person with autoimmune deficiency syndrome (AIDS) Donation of blood to the state agencies Use of public bathrooms in any city

Engaging in unprotected sexual encounters Because HIV is spread through exposure to bodily fluids, unprotected intercourse and shared drug paraphernalia remain the highest risks for this infection. The other actions are not at risk behaviors for HIV.

A client taking isoniazid for tuberculosis (TB) asks the nurse about the side effects of this medication. The client should be instructed to report which of these findings? Photosensitivity and photophobia Confusion and light-headedness Double vision and visual halos Extremity tingling and numbness

Extremity tingling and numbness Peripheral neuropathy is a common side effect of isoniazid and other antitubercular medications and should be reported to the health care provider. Daily doses of pyridoxine (vitamin B6) may lessen or even reverse peripheral neuropathy due to isoniazid use.

A nurse is performing a gestational age assessment on a newborn delivered two hours ago. The nurse is using the Ballard scale. Which of these factors may affect the score? Birth trauma Fetal distress in labor Racial differences Birth weight

Fetal distress in labor The effects of earlier distress may alter the findings of reflex responses as measured on the Ballard tool. Gestational age can be estimated by using other physical characteristics, such as the amount of lanugo, creases on the plantar surface of the feet, ear cartilage, and appearance of the eyes as well as genitalia.

The respiratory technician arrives to draw blood for arterial blood gas (ABG) analysis. What should the nurse understand about the procedure? Firm pressure is applied over the puncture site for at least five minutes after the sample is drawn The femoral artery is the preferred sample site The blood sample must be kept at room temperature and delivered to the lab as soon as possible Supplemental oxygen should be turned off 30 minutes prior to collecting the sample

Firm pressure is applied over the puncture site for at least five minutes after the sample is drawn The radial artery is preferred; the second choice is the brachial artery and then the femoral artery. If a client is receiving oxygen, it should not be turned off unless ordered. After drawing the sample, it's very important to press a gauze pad firmly over the puncture site until bleeding stops or at least five minutes. Do not ask the client to hold the pad because if insufficient pressure is used, a large painful hematoma may form. The sample of arterial blood must be kept cold, preferably on ice to minimize chemical reactions in the blood.

A nurse is caring for a 74 year-old client with benign prostatic hypertrophy (BPH). Which finding would the nurse anticipate when assessing this client? Involuntary voiding with coughing and sneezing Large volume of urinary output with each voiding Urine is dark and concentrated Frequent urination

Frequent urination Clients with BPH have frequent urination in small amounts day and night, with difficulty starting the urinary stream.

A client has received two units of whole blood today after an episode of gastrointestinal bleeding. Which laboratory report should the nurse be sure to monitor closely? Bleeding time Platelets Hemoglobin and hematocrit White blood cells

Hemoglobin and hematocrit The post-transfusion hematocrit provides immediate information about red cell replacement and if there is any continued blood loss; the follow-up hematocrit should be checked around 4 to 6 hours after the infusion is completed.

A pregnant client, at 34-weeks gestation, is diagnosed with a pulmonary embolism (PE). Which of these medications does the nurse anticipate the health care provider will initially order? Subcutaneous heparin 5000 units twice a day Low dose aspirin therapy Heparin infusion to maintain the aPTT at 1.5 to 2 times the control value Warfarin (Coumadin) therapy every other day to maintain a PT at 1.5 to 2 times the control value

Heparin infusion to maintain the aPTT at 1.5 to 2 times the control value Clients diagnosed with PE, whether pregnant or not, are initially treated with intravenous unfractionated heparin (UFH). The client's activated partial thromboplastic time (aPTT) should be monitored and kept in the therapeutic range of between 1.5 to 2 times the baseline value. Alternatively, low molecular weight heparins, such as enoxaparin (Lovenox), can be used to treat PE in women who are pregnant. Warfarin should never be given during pregnancy due to its teratogenic effects. Although aspirin has anticoagulant properties, low dose aspirin therapy (81 mg), with or without heparin, is more often used prophylactically to prevent the development of deep vein thrombosis.

A nurse is teaching home care to the parents of a child diagnosed with acute spasmodic croup. What type of care would be most important to emphasize? Humidified air with an increase in oral fluids Antihistamines to decrease allergic responses Antibiotic therapy for 10 to 14 days Sedation as needed to prevent exhaustion

Humidified air with an increase in oral fluids The most important aspects of home care for a child diagnosed with acute spasmodic croup are humidified air and increased oral fluids. Humidified air helps reduce vocal cord swelling. Taking the child out into the cool night air for 10 to 15 minutes can also reduce night time symptoms. Adequate systemic hydration aids mucociliary clearance by keeping secretions thin and easy to remove with minimal coughing effort.

A group of nurses on a unit are discussing stoma care for clients who have had a stoma made for fecal diversion. Which stomal diversion poses the highest risk for skin breakdown? Sigmoid colostomy Transverse colostomy Ileal conduit Ileostomy

Ileostomy Ileostomy output, which is from the small intestine, is of continuous, liquid nature. This high pH, alkaline output contains gastric and enzymatic agents that when present on skin can denude skin in a few hours. Because of the caustic nature of this stoma output, adequate peristomal skin protection must be delivered to prevent skin breakdown. With a transverse colostomy the stool is of a somewhat mushy and soft nature. With a sigmoid colostomy the output is formed with an intermittent output. An ileal conduit is a urinary diversion with the ureters being brought out to the abdominal wall.

Which statement describes the advantage of using a decision grid to make decisions? It allows the data to be graphed for easy interpretation It is both a visual and a quantitative method of decision making It is the only truly objective way to make a decision in a group It is the fastest way for group decision making

It is both a visual and a quantitative method of decision making A decision grid allows the group to visually examine alternatives and evaluate them quantitatively with weighting.

A young adult seeks treatment in an outpatient mental health center. The client tells the nurse: "I am a government official being followed by spies." On further questioning, the client reveals: "My warnings must be heeded to prevent nuclear war." Which of the following actions should the nurse take? Listen quietly without comment Confront the client's delusion Ask for more information about the spies Contact the government agency

Listen quietly without comment The client's comments demonstrate grandiose ideas. The most therapeutic response is to listen but to also avoid being pulled into the client's delusional system. At some point validation of the present situation will need to be done. Confrontation at this time would be an inappropriate action and is not therapeutic.

A community health nurse has been caring for a 16 year-old who is 22-weeks pregnant with a history of morbid obesity, asthma and hypertension. Which of these lab reports need to be communicated to the health care provider as soon as possible? Hematocrit 33% and platelets 200,000 Magnesium 0.8 mEq/L and creatinine 3 mg/dL Blood urea nitrogen 28 and glucose 225 mg/dL Hemoglobin 11 g/L and calcium 6.7 mg/dL

Magnesium 0.8 mEq/L and creatinine 3 mg/dL The magnesium is low and the creatinine is high, which indicates acute renal failure, which is the highest priority. With the history of hypertension, the findings may indicate preeclampsia. The rest of client's lab values are all abnormal except for the platelets. The client needs to be referred for immediate follow-up with a health care provider.

The parents of a 5 month-old report that the infant has "vomited nine times in the past six hours." Based on this information, the nurse should observe for which fluid and electrolyte imbalance? Metabolic acidosis Metabolic alkalosis Hemoconcentration effects Hemodilution effects

Metabolic alkalosis Vomiting results in a loss of acid from the stomach. Prolonged vomiting results in excess loss of acid and leads to metabolic alkalosis. Findings include irritability, increased activity, hyperactive reflexes, muscle twitching and elevated pulse. Hemoconcentration due to fluid loss may occur, but is not the best answer because it does not answer the question about an imbalance.

The nurse is caring for a client in a violent relationship. The nurse should understand that immediately after an acute battering incident, the batterer may respond to the partner's injuries by taking which action? Seek medical help for the victim's injuries Contact a close friend and ask for help with the incident Be very remorseful and assist the victim to receive medical care Minimize the episode with an underestimation of the victim's injuries

Minimize the episode with an underestimation of the victim's injuries Many batterers lack an understanding of the effects of their behavior on the person who was battered. Batterers use excessive minimization and denial of the situation and their behaviors or intent.

A client has returned from a cardiac catheterization that was two hours ago. Which finding would indicate that the client has a potential complication from the procedure? Decreased urine output Increased heart rate No pulse in the affected extremity Increased blood pressure

No pulse in the affected extremity Loss of the pulse in the extremity would indicate a potential severe spasm of the artery or clot formation to the extent of an occlusion below the site of insertion. It is not uncommon that initially the pulse may be intermittently weaker from the baseline. However, a total loss of the pulse is a nursing emergency. The health care provider needs immediate notification.

A nurse is talking by telephone with a parent of a 4 year-old child who has chickenpox. Which approach demonstrates appropriate teaching by the nurse? The illness is only contagious when the lesions are present Recommend an antiviral medication to relieve itching Chewable aspirin is the preferred analgesic Papules, vesicles and crusts will be present at one time

Papules, vesicles and crusts will be present at one time All three stages of the chickenpox lesions will be present on the child's body at the same time. Children should not be medicated with aspirin due the possibility of developing Reye's syndrome. A person with chickenpox is contagious one to two days before their blisters appear and remain contagious until all the blisters have crusted over. Antiviral medications are not usually prescribed to otherwise healthy children. Over-the-counter hydrocortisone creams can help relieve itchy skin.

The nurse is evaluating a developmentally challenged 2 year-old child. During the evaluation, what goal should the nurse stress when talking to the child's mother? Help the family decide on long-term care Prepare for independent toileting Teach the child self-care skills Promote the child's optimal development

Promote the child's optimal development The primary goal of nursing care for a developmentally challenged child is to promote the child's optimal development.

A newborn who is delivered at home and without a birth attendant is admitted to the hospital for observation. The initial temperature is 95 F (35 C) axillary. The nurse should recognize that cold stress may lead to what complication? Reduced partial pressure of oxygen in arterial blood (PaO2) Lowered basal metabolic rate Metabolic alkalosis Hyperglycemia

Reduced partial pressure of oxygen in arterial blood (PaO2) Hypothermia and cold stress cause a variety of physiologic stresses including increased oxygen consumption, metabolic acidosis, hypoglycemia, tachypnea and decreased cardiac output. The baby delivered in such circumstances needs careful monitoring. In this situation, the newborn must be warmed immediately to increase its temperature to at least 97 F (36 C). Normal core body temperature for newborns is 97.7 F to 99.3 F (36.5 C to 37.3 C).

The nurse, who is located in a large urban area, uses telecommunications to provide health care and education to clients in remote locations. What is the best reason for using telehealth? Reduces health care costs Empowers clients to take a greater interest in their illness Standardizes electronic data sharing of health information Removes time and distance barriers from the delivery of care

Removes time and distance barriers from the delivery of care Telehealth is the use of technology to deliver health care, health information, or health education at a distance. People in rural areas or homebound clients can communicate with providers via telephone, email or video consultation, thereby removing the barriers of time and distance for access to care. Although increased access to information and collaboration between the client and provider can be empowering, this is not the primary reason for using telecommunications/telehealth.

A newly admitted 78 year-old client is diagnosed with severe dehydration. When planning care for this client, the nurse should assign which task to an unlicensed assistive personnel (UAP)? Report hourly outputs of less than 30 mL/hr within 15 minutes of the check Converse with the client to determine if the mucous membranes are impaired Check skin turgor every four hours along with the need to change the adult diaper Monitor client's ability for movement in the bed from side to side

Report hourly outputs of less than 30 mL/hr within 15 minutes of the check When assigning a UAP, the nurse must communicate clearly about each delegated task with specific instructions on what must be reported and when. Because the RN is responsible for all care-related decisions, only routine tasks should be assigned to UAPs because such tasks do not require judgments and decisions.

A nurse is reviewing laboratory results on a client diagnosed with acute renal failure. Which lab result should be reported immediately? Venous blood pH 7.30 Hemoglobin of 9.3 mg/dL Serum potassium 6 mEq/L Blood urea nitrogen 50 mg/dL

Serum potassium 6 mEq/L Although all of these findings are abnormal, the elevated potassium level is a life threatening finding and must be reported immediately. Normal serum levels of potassium are 3.5 to 5.0 mEq/L

Nursing students are reviewing the various types of oxygen delivery systems. Which oxygen delivery system is the most accurate? A nasal cannula A partial nonrebreather mask The simple face mask The Venturi mask

The Venturi mask The most accurate way to deliver oxygen to the client is through a Venturi system such as the Venti Mask. The Venti Mask is a high flow device that entrains room air into a reservoir device on the mask and mixes the room air with 100% oxygen. The size of the opening to the reservoir determines the concentration of oxygen. The client's respiratory rate and respiratory pattern do not affect the concentration of oxygen delivered. The maximum amount of oxygen that can be delivered by this system is 55%.

The charge nurse has a health care team that consists of one practical nurse (PN), one unlicensed assistive personnel (UAP) and one PN nursing student. The charge nurse has made these assignments. Which assignment should be questioned by the nurse manager? The admission at the change of shifts of a client diagnosed with atrial fibrillation and acute heart failure - PN An older adult client who had been diagnosed with a myocardial infarction one week ago - UAP A child diagnosed with second-degree burns over 30% of the body and has IV packed red cells running and an order for albumin IV - charge nurse A client who was diagnosed with a major stroke six days ago - PN nursing student

The admission at the change of shifts of a client diagnosed with atrial fibrillation and acute heart failure - PN The care for a new admission with risk of instability should be performed by an RN. In addition, because the client was admitted at the change of shifts, the stability of the client would not have been established. The charge nurse should take this client. PNs should be delegated clients who are stable with minimal risk for instability; medication administration, except for IV push; sterile procedures; reinforcement of teaching and other client care that does not require judgments and decisions. The nurse may delegate routine tasks and a stable client to UAPs.

During a situation of pain management, which statement is a priority to consider for the ethical guidance of a nurse? Clients have the right to have their pain relieved The client's self-report is the most important consideration Cultural sensitivity is fundamental to pain management Nurses should not prejudge a client's pain using their own values

The client's self-report is the most important consideration Pain is a complex phenomenon that is perceived differently by each individual. Pain is whatever the client says it is. The other statements are correct but not the most important consideration.

An 18 month-old child is on peritoneal dialysis in preparation for a renal transplant in the near future. When a nurse obtains the child's health history, the mother indicates that the child has not had the first measles, mumps, rubella (MMR) immunization. What should the nurse understand about the child's situation and administration of the immunization? An inactivated form of the vaccine can be given at any time Live vaccines are withheld in children with renal chronic illness The measles, mumps and rubella (MMR) vaccine should be given now, before the transplant The risk of the vaccine side effects precludes the administration of the vaccine

The measles, mumps and rubella (MMR) vaccine should be given now, before the transplant MMR is a live virus vaccine, and should be given at this time. Post-transplant, immunosuppressive drugs will be given and the administration of the live vaccine at that time would be contraindicated because of the compromised immune system.


Related study sets

LAP: Can you relate? - Student Guide

View Set

Chapter 8 Intro To Business Study Guide

View Set

Chapter 27: Patient Safety and Quality

View Set

Database Concepts Test #1 chapter 1 & 2

View Set